« first day (1892 days earlier)      last day (3123 days later) » 
00:00 - 15:0015:00 - 00:00

3:01 PM
If anyone cares:
http://tex.stackexchange.com/a/402
That is what I decided was best after a few downloads.
 
3:16 PM
What does 'leaving pointwise fixed some hyperplane' mean? Specifically 'pointwise', I don't see why 'leaving some hyperplane fixed' would not do
(referring to a reflection)
 
3:28 PM
Does anyone know a notation for $f:X\rightarrow Y$ that also suggests that $f(A)\subseteq B$ for some $A\subseteq X$ and $B\subseteq Y$
 
Will $f|_{A}$ suffice?@Alec
 
@Rememberme I want a notation that suggests that, yes.
For example $f:[X,A]\rightarrow [Y,B]$ - I just made this up, but could denote that $f$ restricted to $A$ has image $\subseteq B$
 
@AlecTeal $f:X\to Y$ will do
 
The incredibly ugly $f: A\subset X \to B\subset Y$?
 
@LieAlgebra that doesn't suggest (or even mention) anything about $A$ and $B$
@Krijn have you ever seen that before anywhere? (I like that as a falback)
 
3:32 PM
@AlecTeal It says that $A\hookrightarrow X \to Y$
 
I haven't seen that before, but I have a professor that denotes these beneath each other
So
$f: X \to Y$
$A \to B$
 
And clearly then $f(A)\subseteq Y$
 
But thats on a blackboard, which is easier than LaTeX
 
$f:\overset{X\to Y}{A\to B}$
 
With small $\subset$'s between $A$ and $X$ would be nice
 
3:35 PM
Like facing downwards?
upwards*
 
Well, upwards, I guess
Yeah
 
$f:\left(\begin{matrix}X&\to&Y\\\cup&&\cup \\A&\to&B\end{matrix}\right)$
Well that didn't work lol
$$f:\left(\begin{matrix}X&&Y\\\cup&\to&\cup \\A&&B\end{matrix}\right)$$

I'm done
 
@LieAlgebra I appreciate it but I can invent stuff myself.
 
This last one does certainly resemble how my teacher would write it
It is not hard to prove that $a+b | a^n + b^n$ if $n$ is odd. Is there some nice interpretation of this fact?
 
3:51 PM
@Huy Are you in Switzerland by any chance ?
 
Yo guys.
 
@Hippalectryon Huy is in ETH Zurich
 
Ah, makes sense
 
@LieAlgebra please stop.
PLEASE
 
@AlecTeal ? I stopped long ago? 19 minutes
@AlecTeal CLARIFY?
@AlecTeal $$f: \begin{pmatrix}X\\\cup\\A\end{pmatrix}\to \begin{pmatrix}Y\\\cup\\ B\end{pmatrix}$$
 
4:05 PM
@Hippalectryon Why did you ask?
 
@AlecTeal $f:X\to Y$ is probably best, since what you are after is entirely implicit, since $B=Y$ has $B\subseteq Y$.
 
@Rememberme He gave me a pdf of something you've unlikely yo find by yourself on the internet, and in the Mendley database a significant part of the ones who owned that pdf were from Switzerland
 
Ahh, makes sense why you ask
 
Maybe I forgot my derivation, but if I wanted to find the derivative of $y = f(x)$ with respect to $x$ (assuming derivatives do exist for $f(x)$, $x \in \mathbb{R}$), then I would notationally represent it as

$$
\frac{d}{dx}y = \frac{d}{dx}f(x)
$$
Correct?
 
Correct, but more often, if it is clear that we are deriving with respect to $x$ we would just write $f'(x)$
 
4:09 PM
I have to go now @AlecTeal, let me know if you need further help
I have a lecture to attend
 
I'm asking because I came across the $\\nabla$ notation, which represents a gradient. Maybe I read wrong, but that is used to express the "derivative" of some $f:\mathbb{R}^n \to \mathbb{R}^n$, e.g. the derivative of $f(\mathbf{x})$, $\mathbf{x} \in \mathbb{R}^n$.
Did I get it correct so far?
By $\\nabla$ I meant $\nabla$.
I'm asking because I came across this equation, and I'm asked to solve for $\mathbf{w}$, in the following equation:

$$
E(\mathbf{w}) = \frac{1}{2}\sum\limits_{i = 1}^N(t_n - \mathbf{w}^{\text{T}}\phi(\mathbf{x}_n))^2 + \frac{\lambda}{2}\mathbf{w}^{\text{T}}\mathbf{w}
$$
 
Huy
@Hippalectryon yeah, at ETH, why?
 
14 mins ago, by Hippalectryon
@Rememberme He gave me a pdf of something you've unlikely yo find by yourself on the internet, and in the Mendley database a significant part of the ones who owned that pdf were from Switzerland
 
It's suggested to find the derivative, and then set the derivative of $E(\mathbf{w}) to 0, and solve for $\mathbf{w}$.
 
Huy
Which one? Do you mean the functional analysis textbook draft?
 
4:21 PM
yes. Surprisingly, several people have it on mendley
 
Huy
Doesn't have to be surprising. You can find it via the website from Prof. Einsiedler and he isn't that unknown
 
Oh, ok
 
Huy
And I can imagine the coauthor also distributes it to students / friends
I find it quite interesting so far because it offers many different motivations and doesn't just touch the surface of many things
So you're using mendeley and not readcube @Hippalectryon ?
 
@SalehenRahman Basically gradient of a $f, f:\Bbb{R^m}\to \Bbb{R^n}$ is a vector field at each point $a$ where partial derivatives $D_1(f(a),....D_n(f(a))$ exist. To get an idea of this in $\Bbb{R^2}$ $\nabla f(a) = \frac{{\partial}(a)}{{\partial}{x}}\hat{i} + \frac{{\partial}(a)}{{\partial}{y}}\hat{j}$
 
@Huy Well after our discussion yesterday I thought I'd try out others, and Mendley takes clearly less processor
 
Huy
4:30 PM
Yeah I did a bit of research and found out mendeley was the standard thing for what you do with readcube
Might try it out after dinner
 
@LieAlgebra please do not return.
 
4:45 PM
Any tips for increasing mental math speed? Or just the good ole practice method?
 
5:02 PM
it's for an interview just to be clear... not trying to set any records for the "mad minute".
 
I don't know any method for that other than practicing
 
Yeah, that's what I assumed for multiplication/division. Series are tough, IMO.
 
6:12 PM
@hippa Why is your showcase closed ?
@hippa I can't give up
 
@Ramanewbie Give up on giving up
 
@hippa No, because if I do, I won't be able to give up for sure !
 
Hi!!! Could I ask you something ? Is the curve $\sin t \cos t$ the same as $- \sin t \cos t$ with the only difference their direction?
 
@hippa Do you have another open place
 
@Ramanewbie no
@evinda How do you define the direction of a curve ?
 
6:14 PM
@hippa Why is it closed then ? (Because I won't give up on giving up)
 
@Ramanewbie No idea why. I don't even want to know why.
 
We haven't defined it. After some operations at an exercise I got $y= \pm \cos t \sin t$ and I don't know why we reject $-$ :/ @Hippalectryon
 
@hippa Why you not open it.
I can't give up nor see meteo
 
Are you familiar with differential geometry? @Hippalectryon
 
@evinda Just a bit. Ask anyway.
 
6:19 PM
@Hippalectryon My research reveals again amazing results ...
 
@Chris'ssistheartist :D
 
@Hippalectryon Firstly I have showed that $r(t)=(\cos^2 t-\frac{1}{2}, \sin t \cos t, \sin t)$
satisfies both of the given surfaces $x^2+y^2=\frac{1}{4}$ and $\left( x+\frac{1}{2}\right)^2+y^2+z^2=1$ and hence their intersection, that is $x^2+x=\frac{1}{2}$.

It remains to show that any point on that intersection is of the form $(\cos^2 t-\frac{1}{2}, \sin t \cos t, \sin t)$.

If $(x,y,z)$ lies on the sphere $\left( x+\frac{1}{2} \right)^2+y^2+z^2=1$ then $z^2=1-\left( x+\frac{1}{2} \right)^2-y^2 \leq 1$. Therefore $|z| \leq 1$. It follows that $z$ must be of the form $z= \sin t$ for s
 
@Hippalectryon :D
 
@evinda "Substituting z=sint into the equation $z^2+x=1/2$" isn't it $x^2+x=1/2$ ?
 
@Hippalectryon No, the intersection is $z^2+x=\frac{1}{2}$, it was a typo.. Or am I wrong?
@Hippalectryon $x^2+y^2=\frac{1}{4}$, $\left( x+\frac{1}{2}\right)^2+y^2+z^2=1$
Isn't the intersection $z^2+x=\frac{1}{2}$ ?
 
6:29 PM
Yes, that's right
Shouldn't $\sin^2 t+x=\frac{1}{2} \Rightarrow x= \cos^2 t-1$ be $\sin^2 t+x=\frac{1}{2} \Rightarrow x= \cos^2 t-1/2$ ?
 
x=1/2-sin^2 t=1/2-(1-cos^2 t)=cos^2 t-1/2 @Hippalectryon
Yes, sorry, again a typo... That's what we want to get...
 
@evinda Well what you have obtained is a necessary condition on $y$, not a sufficient one, right ?
 
@Hippalectryon How can we find a sufficient?
 
@evinda You know that any point on the intersection is either $(\cos^2t-1,\cos t\sin t,\sin t)$ or $(\cos^2t-1,-\cos t\sin t,\sin t)$. Now check whether both those cases actually work.
 
The equations that contain a have y^2, so we can't reject -\cos t\sin t. Or am I wrong?
 
6:41 PM
@evinda Well first of all, what is the problem exactly ? You've shown me your reasoning, but I don't know your goal.
 
I want to show that y= sintcost and not +- sintcost @Hippalectryon
 
@evinda In your reasoning, you talk of $r(t)$. Where does it come from ? When you later say 'it remains to show', what is that based on ? WHat's the more global goal of the exercise ?
 
That is the exercise that I am given:

Show that $r(t)=\left (\cos^2 t-\frac{1}{2}, \sin t\cos t, \sin t\right )$ is a parametrization of the curve
of intersection of the circular cylinder of radius $\frac{1}{2}$ and axis the $z$-axis with the sphere of radius $1$ and centre $\left (-\frac{1}{2}, 0, 0\right )$. This is called Viviani’s Curve. @Hippalectryon
 
$\dfrac{\bar{\Xi}}{\Xi}$
ugh
 
@SohamChowdhury Oh please :( not that awful notation
 
6:47 PM
Who shows me a brilliant evaluation of $$\int _0^1\int _0^1\int _0^1\frac{\log (x+y+z)}{1+x y z} \ dx \ dy \ dz$$ ?
 
@evinda By any chance, aren't the two curves (the one with the + and the one with the -) the same curve ? Did you check symmetries ?
 
How could we justify it formally that the two cures are the same curve?
 
By showing that for any image $c_1(t)$ there exists $t_2$ such that $c_2(t_2)=c_1(t)$ where $c_1,c_2$ are the two curves
 
@Hippalectryon Whom should I ask?
 
@Chris'ssistheartist Well, MNCE isn't here today :(
 
6:52 PM
I see.
 
For any $c_1(t)= \sin t \cos t$ there exists $t_2$ auch that $c_2(t_2)=- \sin{t_2} \cos{t_2}= \sin t \cos t$.
We can pick $t_2=-t$. Then $c_2(t_2)=-\sin (-t) \cos (-t)= \sin t \cos t$. @Hippalectryon
Is it like that? @Hippalectryon
 
Well, it has to work for the other two coordinates too. (which it maybe does, I didn't check)
 
You mean that it has to hold that x(t)=x(-t) and z(t)=z(-t) ? @Hippalectryon
 
yes. (which doesn't work since z(-t)=-z(t)
)
However, what about $t_2=\pi-t$ ?
 
@SohamChowdhury It looks awesome on a blackboard. I have tried it.
 
6:58 PM
let's say that I have a number C which will be hypotenuse of pythogorean triple. I need to find a,b such that a^2 + b^2 = c^2 and both a and b are integers.
 
So is the following justification right and sufficient? Or could we improve something?
For any $c_1(t)= \sin t \cos t$ there exists $t_2$ auch that $c_2(t_2)=- \sin{t_2} \cos{t_2}= \sin t \cos t$.
We can pick $t_2=\pi-t$. Then $c_2(t_2)=-\sin (\pi-t) \cos (\pi-t)= - \sin t (- \cos t)= \sin t \cos t$, $\cos^2(\pi-t)-\frac{1}{2}= \cos^2 t-\frac{1}{2}, \sin(\pi-t)=\sin t$. @Hippalectryon
 
@evinda I just realized I told you something that was wrong before. The existence of $t_2$ is not enough. However, here we have shown that for any $t$, $c_2(t)=c_1(\pi-t)$. As a result, the set of images of both curves are exactly identical. Hence the curves are the same (as in, their 3D representation is the same).
 
Today I prepared a challenge for the kids I train: Calculate by polar coordinates $$\int_0^1 \frac{\log^2(1+x)}{x} \ dx$$ together with a hint.
 
@hippa Still not working :(
 
@Ramanewbie Do @Chris'ssistheartist's integral above and I'll open it :D
 
7:10 PM
@hippa Alright
 
@Chris'ssistheartist is that log(log(1+x)) or log((1+x)^(1+x))
 
@AlecTeal Neither one of them. It's $(\log(1+x))^2=\log(1+x)\times \log(1+x)$.
 
@Hippalectryon So can we justify it as follows? Or can we say it better?
Since $- \sin t \cos t=\sin (\pi-t) \cos (\pi-t)$, $x(t)=x(\pi-t)$, $z(t)=z(\pi-t)$ we have that $y=\sin t \cos t$.
 
@hippa The solution is$$x}\right)\ln^{2}\left(x+1\right)+2\operatorname{Li}_{2}\left(x+1\right)\ln\l‌​eft(x+1\right){-2\operatorname{Li}_{3}\left(x+1\right)}+C$$
 
@evinda sure
@Ramanewbie ಠ_ಠ what about 1) no, 2) I said do the integral not ask wolfram for the answer
 
7:19 PM
@hippa Then I can't do it
@hippa I don't even know what 'do' an integral means
 
compute
 
@hippa Calculate ?
 
@Hippalectryon Nice, thank you :)
 
@hippa I don't know what it means nor how to proceed.
Am I supposed to be able to compute it.
 
7:22 PM
@Chris'ssistheartist What's the age of your students ?
 
@Hippalectryon 16-18 these days
 
@Chris'ssistheartist can I have a hint?
 
bears
polar bears
 
@AlecTeal Yes, but an incomplete one. Write that integral as a double integral, in an useful way.
 
@hippa ^
 
7:31 PM
Don't imagine you find a double integral and you're done. You need some clever tricks there.
 
7:46 PM
Here is also an inspirational song:
 
@Chris'ssistheartist Are you a maths teacher now ?
 
@Ramanewbie I attend tutoring (amongst other things). No, I'm not a teacher.
 
@Chris'ssistheartist amongst raising dogs ? ;)
 
@Ramanewbie hehe, also that one :D
 
hi @Chris'ssistheartist @Hippalectryon@Ramanewbie
 
7:48 PM
Hi @gato
 
How do we calculate upper and lower riemann sums for a given partition?
 
@Gato hiii :D
 
@Hippalectryon it's been a long time
 
Indeed
 
7:50 PM
how are you ?
what's up ?
:)*
 
Well, chem exam tomorrow ;(
@Chris'ssistheartist Hmm... I'm starting to think that there's a way to solve that integral using Euler's biased summation methods :P
 
isn't your favorite course ? :p
 
@Gato Not really >:c
 
@hippa You have a test again ??
 
@Hippalectryon I'm not familiar with that description. Euler–Maclaurin formula?
 
7:53 PM
@Paradox101 What exactly are you working on? A proof or a particular calculation?
 
@Chris'ssistheartist I'm talking about the ones that leads to $\zeta(-1)=-1/12$ for instance
@Ramanewbie every week
 
@Hippalectryon Ah, that is a different stuff, analytic continuation of Riemann zeta function.
 
@hippa Oh I forgot tomorrow was Saturday the week went fast.
 
@Chris'ssistheartist Basically $\int_0^1\ln^2(1+x)/xdx=\int_0^1\sum_{i,j=1}^\infty\dfrac{(-x)^{i+j-1}}{ij}=\sum‌​_{i,j=1}^\infty(-1)^{k+j}$... which doesn't converge in the usual sense, but we might be able to do something with it
 
@Hippalectryon tu connais des problèmes sur les groupes ?
 
7:56 PM
@Hippalectryon It's not pleasant (at first sight) ... :D
 
@Gato Hmm.... pas vraiment sur les groupes, mais ça me fait penser à l'exo suivant : soit $C_n$ l'ensemble des matrices de $\mathcal{M}_n(\mathbb{R})$ tq $^tAA=I_n$, on note $d_n$ la dimension maximale d'un sous espace vectoriel de $C_n$. donner $d_1,d_2,d_3$.
 
@Hippalectryon In the meantime I admire my solution in polar coordinates. :D
 
@Ramanewbie Sanic is always fast
 
@hippa Then I don't know how he did it but Sanic managed to make my professors teach me some interesting stuff this week so it went fast...
 
@Hippalectryon merci, pas mal l'exo
 
8:00 PM
@Gato Aparemment ça vient d'un oral d'ENS
 
@Addem I'm working more on a computation than a proof. A function and its partition is given and we are supposed to find the upper and lower sums for that partition
 
So it sounds like for the lower sum you would use the lowest end-points of each partition, plug them into the function and collect a bunch of values. Pair them with the partition lengths and multiply pairwise. This results in a new list of values which you then sum. @Paradox101
 
@Hippalectryon ah oui j'ai pas regardé les oraux de cette année
 
@Addem if for instance i have a function $f=x$ with partition $P={0,1,2,3,4,...}$ then the lower sum would be 0$f(0)*1+f(1)*1+f(2)*1+...$?
 
@Paradox101 Yes, which would obviously not converge, which just means that this particular sum is not well-defined. That's not surprising since the area under the curve to infinity is not defined for this function. But that would be basically the procedure.
 
8:08 PM
@Chris'ssistheartist Was the hint the nice series for $\log(1+x)^2$?
 
@Addem Yes the sum isn't well defined, I just made it up to get the concept. thanks a lot! :)
 
@robjohn No. Indeed, there is a nice series in harmonic numbers.
 
@Addem can I ask you another question related to partition?
 
Shoot.
 
@Chris'ssistheartist The integral is the same as $$\sum_{k=1}^\infty\frac{H_k}{(k+1)^2}$$ and that can be simplified.
 
8:11 PM
 
@robjohn Yeah.
 
@Chris'ssistheartist To $\zeta(3)$
 
@Addem I solved this by computing the riemann sum using the lower tags, $f(1.5)*0.5+f(2.4)*0.9+..$. Is this method correct?
 
@robjohn Yeap.
 
@Addem I checked the answer with wolframalpha for another part to this question but my answer was very different.
 
8:16 PM
Yeah, this seems right, assuming that the partition is a partition of the interval [1, 3]. How would you enter this into Wolfram?
Oh yeah, now I see the instructions that it's on [1, 3] so this is correct.
 
@Addem I just entered riemann sum followed by the function and the interval. In the detailed answer they mentioned whether it was using left or right endpoints
 
I'm not exactly sure what you mean by entering the Riemann sum but there's a chance Wolfram is just assuming that you want the exact integral, not an approximation by this particular partition.
I can't speak to what Wolfram is telling you without knowing what you entered, but the pattern you gave above, f(1.5)*0.5+f(2.4)*0.9+.., is correct.
 
Hi, one question. Is there any simple way to formally express a disjoint set?
The definition of a pairwise disjoint set $\mathbb{S}$ is the following.
$$\forall X, Y \in \Bbb S: X \ne Y \implies X \cap Y = \varnothing$$
 
@Hippalectryon this might help math.stackexchange.com/questions/1091812
 
What I'm looking for is some simple formula $\phi(\mathbb S)$ such that
$$\phi(\mathbb S)\iff\left[\forall X, Y \in \Bbb S: X \ne Y \implies X \cap Y = \varnothing\right].$$
 
8:30 PM
@Hippalectryon I missed the alternating sign in what I said above, and that accounts for the factor of $\frac14$ evidently.
 
@Addem Ok thank you so much for your help :)
 
Or I can also ask like this : Is there any symbolic notation for disjoint sets?
 
@Paradox101 No problem.
 
@robjohn Thanks, but I was wondering if one could get the sale result using the improper sum I got above
 
8:55 PM
If a finite set $\mathbb{S}$ is disjoint, then $\left|\bigsqcup\mathbb{S}\right|=\left|\bigcup\mathbb{S}\right|$ holds. Does this also hold for infinite case?
 
9:23 PM
hello!
 
 
1 hour later…
10:38 PM
Howdy
 
@Pies ಠ_ಠ
 
Oh hey
 
Do you have any math puns today
 
I'd rather not go off on a tangent, cos that won't be a good sine
 
10:45 PM
lol
 
...I got no life
 
Do you study the whole day? @Pies
 
Of course
@Pies is the best student
 
Study what?
 
Nice
@Pies mathematics
 
10:52 PM
Physics
 
Not really, more so astronomy and physics
 
Aha @Pies
How old are you? @Pies
 
17, you?
 
20
 
._.
 
10:53 PM
@Pies Where are you from?
 
Er...the US
 
I will go off now. Good night!!!
 
'night
 
11:35 PM
Do we party now that M.SE has reached 500K questions?
 
11:48 PM
Wow
 
I've written answers for 0.4% of the questions.
 
@ThomasAndrews are there 250 questions?
 
00:00 - 15:0015:00 - 00:00

« first day (1892 days earlier)      last day (3123 days later) »